Difference between revisions of "2006 AIME I Problems/Problem 14"

(Solution)
m (cat)
Line 1: Line 1:
 
== Problem ==
 
== Problem ==
 
+
A tripod has three legs each of length 5 feet. When the tripod is set up, the angle between any pair of legs is equal to the angle between any other pair, and the top of the tripod is 4 feet from the ground In setting up the tripod, the lower 1 foot of one leg breaks off. Let <math> h </math> be the height in feet of the top of the tripod from the ground when the broken tripod is set up. Then <math> h </math> can be written in the form <math> \frac m{\sqrt{n}}, </math> where <math> m </math> and <math> n </math> are positive integers and <math> n </math> is not divisible by the square of any prime. Find <math> \lfloor m+\sqrt{n}\rfloor. </math> (The notation <math> \lfloor x\rfloor </math> denotes the greatest integer that is less than or equal to <math> x. </math>)
Let <math> S_n </math> be the sum of the reciprocals of the non-zero digits of the integers from 1 to <math> 10^n </math> inclusive. Find the smallest positive integer n for which <math> S_n </math> is an integer.
 
  
 
== Solution ==
 
== Solution ==
Let <math>K = \sum_{i=1}^{9}{\frac{1}{i}}</math>.  Examining the terms in <math>S_1</math>, we see that <math>S_1 = K + 1</math> since each digit <math>n</math> appears once and 1 appears an extra time.  Now consider writing out <math>S_2</math>.  Each term of <math>K</math> will appear 10 times in the units place and 10 times in the tens place (plus one extra 1 will appear), so <math>S_2 = 20K + 1</math>.
+
{{solution}}
 
 
In general, we will have that:
 
 
 
<math>S_n = (n10^{n-1})K + 1</math>
 
 
 
because each digit will appear <math>10^{n - 1}</math> times in each place in the numbers <math>1, 2, \ldots, 10^{n} - 1</math>, and there are <math>n</math> total places. 
 
 
 
The denominator of <math>K</math> is <math>D = 2^3\cdot 3^2\cdot 5\cdot 7</math>.  For <math>S_n</math> to be an integer, <math>n10^{n-1}</math> must be divisible by <math>D</math>.  Since <math>10^{n-1}</math> only contains the factors 2 and 5 (but will contain enough of them when <math>n \geq 3</math>), we must choose <math>n</math> to be [[divisible]] by <math>3^2\cdot 7</math>.  Since we're looking for the smallest such <math>n</math>, the answer is <math>063</math>
 
  
 
== See also ==
 
== See also ==
 
{{AIME box|year=2006|n=I|num-b=13|num-a=15}}
 
{{AIME box|year=2006|n=I|num-b=13|num-a=15}}
 +
 +
[[Category:Intermediate Geometry Problems]]

Revision as of 19:35, 25 September 2007

Problem

A tripod has three legs each of length 5 feet. When the tripod is set up, the angle between any pair of legs is equal to the angle between any other pair, and the top of the tripod is 4 feet from the ground In setting up the tripod, the lower 1 foot of one leg breaks off. Let $h$ be the height in feet of the top of the tripod from the ground when the broken tripod is set up. Then $h$ can be written in the form $\frac m{\sqrt{n}},$ where $m$ and $n$ are positive integers and $n$ is not divisible by the square of any prime. Find $\lfloor m+\sqrt{n}\rfloor.$ (The notation $\lfloor x\rfloor$ denotes the greatest integer that is less than or equal to $x.$)

Solution

This problem needs a solution. If you have a solution for it, please help us out by adding it.

See also

2006 AIME I (ProblemsAnswer KeyResources)
Preceded by
Problem 13
Followed by
Problem 15
1 2 3 4 5 6 7 8 9 10 11 12 13 14 15
All AIME Problems and Solutions